If a manufacturer's list price is $800 and Bill's Outlet buys the goods for $650, what is the trade discount percent? (Round answer to
the nearest hundredth percent)

Answers

Answer 1

Answer:

The discount percent is 18.75%

Step-by-step explanation:

The amount of money that was lost in this discount is $150, to calculate the percent, you subtract the original price by the discounted price, and then divide the difference by the original price and multiply that by 100% to get the discount percent. [tex]\frac{150}{800}[/tex] * 100% equals 18.75%, so that would be the discount percent.


Related Questions

Find the greatest common factor of 15n³ and 9n.
11
9
X
3

Answers

physical sciences 2023

Rhett made a simple drawing of his house. It is a three dimensional figure with four faces that are rectangular and two that are square what kind of figure is it?

Answers

Rhett's house is a rectangular prism

Given data ,

Let the figure be represented as A

Now , the value of A is

Rhett made a simple drawing of his house.

And , It is a three dimensional figure with four faces that are rectangular and two that are square

So , the figure is represented as a rectangular prism

A three-dimensional solid object called a prism has two identical ends. It consists of equal cross-sections, flat faces, and identical bases. Without bases, the prism's faces are parallelograms or rectangles.

Hence , the figure is rectangular prism

To learn more about volume of prism click :

https://brainly.com/question/11336446

#SPJ1

A ladder is leaning against a building, forming a 70° angle with the ground: The base of the ladder is 8.2 ft from the base of the
building.
What is the length of the ladder?
Round your answer to the nearest tenth of a foot.

22.5 ft
24.0 ft
28.0 ft
28.7 ft

Answers

The length of the ladder that is leaning on the building would be = 24ft. That is option B.

How to determine the length of the ladder?

To determine the length of the ladder, the sine rule needs to be obeyed. That is

= a/sinA = b/sinB

Where;

a = 8.2 ft

A = 180-( 70+90

= 180- 160

= 20°

b = X

B = 90°

That is;

8.2/sin20° = b/sin90°

Make b the subject of formula;

b = 8.2×1/0.342020

= 23.9

= 24 ft

Learn more about length here:

https://brainly.com/question/29882625

#SPJ1

2. Find the amount of tax owed: commercial property valued at $340,000.00; assessed at 25%; taxed at $14.00 per hundred
dollars worth of property.

Answers

Answer:

55.0ml is answer = 22.05

Step-by-step explanation:

A ramp has a vertical rise of 4 feet & the angle of elevation of the ramp is 12°. How long is the ramp?

Answers

Answer:

19.24 ft

Step-by-step explanation:

x = length of ramp

sin12 = 4/x

x = 4/sin12 = 19.24 ft

For a charity drive, each classroom is given a coin box made of cardboard like the one shown. The student council wants to construct a version of the coin box that has a scale factor of 3 times the classroom coin box. Is 100 square feet of cardboard enough to build the new coin box? Select the response with the correct answer and an argument that can be used to defend the solution

Answers

The classroom coin box required less than 11.11 Square feet of cardboard (100 square feet divided by 9), then 100 square feet would be sufficient

The 100 square feet of cardboard is enough to build the new coin box with a scale factor of 3 times the classroom coin box, we need to consider the relationship between the areas of the two boxes.

The area of the original classroom coin box is unknown, as it is not given in the problem. However, since we know that the new coin box has a scale factor of 3 times the original, we can infer that its area will be 3^2 = 9 times greater than the original.

Let's assume that the area of the classroom coin box is A square feet. In that case, the area of the new coin box would be 9A square feet.

Now, we are given that 100 square feet of cardboard is available. To determine if it's sufficient, we need to compare this value to the area of the new coin box, which is 9A.

If 100 square feet is greater than or equal to 9A, then it would be enough to build the new coin box. Conversely, if 100 square feet is less than 9A, it would not be enough.

Since we do not have a specific value for A, we cannot determine the exact answer. However, we can provide the argument to defend our solution.the classroom coin box required less than 11.11 square feet of cardboard (100 square feet divided by 9), then 100 square feet would be sufficient.

To know more about Square feet.

https://brainly.com/question/29020717

#SPJ11

Note the question be like :

A bag contains 5 red marbles, 3 blue marbles, and 2 green marbles. If two marbles are drawn from the bag without replacement, what is the probability that both marbles are red?

A sequence can be generated by using an= 3an-1, where a1 = 10 and n is a whole number greater than 1.
What are the first 3 terms in the sequence?
A. 3, 13, 23
B. 10, 30, 90
C. 10, 13, 16
D. 3, 30, 300

Answers

Answer:

B

Step-by-step explanation:

using the recursive rule [tex]a_{n}[/tex] = 3[tex]a_{n-1}[/tex] and a₁ = 10, then

a₁ = 10

a₂ = 3a₁ = 3 × 10 = 30

a₃ = 3a₂ = 3 × 30 = 90

the first 3 terms are 10 , 30 , 90

Please help quick! will mark brainliest!

Answers

Answer:

7, 14

Step-by-step explanation:

Look at the first column of numbers: 2.8 m in 2 min.

We can get a unit rate from those numbers:

2.8 m / 2 min = 1.4 m/min

He walks at a rate of 1.4 m/min

Now multiply the unit rate by each number of minutes to find the number of meter.

Let's do the second column as a check: time = 3 min

1.4 m/min × 3 min = 4.2 m

4.2 m is the number on the table, so our unti rate is correct.

5 min:

1/4 m/min × 5 min = 7 m

10 min:

1.4 m/min × 10 min = 14 m

Find domain+range
Thank you for the help :-)

Answers

Answer:

Domain: {-15, 0, 9, 33, 44, 45}

Range: {15, 21, 33, 46, 57, 97}

Step-by-step explanation:

The domain and range describe the x and y-values of a function.

Domain

The domain of a function shows the x-values covered by a function. Since the function cannot be proved to be continuous, we have to list the domain as a discrete list of values. We cannot say that the values between the listed x-values are a part of the domain. Any x-value included in the function is a part of the domain. As stated in the question, we should list our answer as an ordered list with braces.

Domain: {-15, 0, 9, 33, 44, 45}

Range

The range of a function shows the y-values covered by the function. Similar to the domain, we cannot assume that the y-values between those listed are included in the range. This means that the range will also be a list of values. Range and domain should be listed from least to greatest. So, we can create a list of all the y-values in numerical order.

Range: {15, 21, 33, 46, 57, 97}

The diagram represents the trajectory of an airplane at take off. The angle that represents the
trajectory of a jet is 15% greater than the trajectory of the airplane.

Answers

The diagram represents the trajectory of an airplane at take off the angle representing the airplane's trajectory is approximately 147.83 degrees.

To solve this problem, let's denote the angle of the airplane's trajectory as x degrees. According to the given information, the angle representing the trajectory of the jet is 15% greater than the airplane's trajectory.

The angle representing the trajectory of the jet can be calculated as:

x + 0.15x = 1.15x

Therefore, the angle representing the trajectory of the jet is 1.15x degrees.

Given that the airplane's trajectory is at 170 degrees, we can set up the following equation:

1.15x = 170

To solve for x, divide both sides of the equation by 1.15:

x = 170 / 1.15 ≈ 147.83

Thus, the angle representing the airplane's trajectory is approximately 147.83 degrees, and the angle representing the jet's trajectory is approximately 1.15 * 147.83 ≈ 170 degrees.

For more details regarding trajectory, visit:

https://brainly.com/question/88554

#SPJ1

Solve and graph. |2x(x-1)+12|<=18

Answers

The solution of the inequality is -1 ≤ x ≤ 3.

To solve the inequality |2x(x-1)+12| ≤ 18, we can break it down into two separate inequalities:

2x(x-1) + 12 ≤ 18

-(2x(x-1) + 12) ≤ 18

Solving the first inequality:

2x(x-1) + 12 ≤ 18

2x² - 2x + 12 ≤ 18

2x² - 2x - 6 ≤ 0

Solving the second inequality:

-(2x(x-1) + 12) ≤ 18

-2x(x-1) - 12 ≤ 18

-2x² + 2x - 12 ≤ 18

2x² - 2x + 30 ≤ 0

Now, let's solve each inequality separately.

For the first inequality, 2x² - 2x - 6 ≤ 0:

We can factor this quadratic equation:

(2x + 2)(x - 3) ≤ 0

To determine the sign of the inequality, we need to consider the intervals where the expression is positive or negative. We set each factor equal to zero to find the critical values:

2x + 2 = 0 => x = -1

x - 3 = 0 => x = 3

Plotting these critical values on a number line and testing intervals, we find that the solution is -1 ≤ x ≤ 3.

For the second inequality, 2x² - 2x + 30 ≤ 0:

This quadratic equation does not have real solutions, so there are no solutions to this inequality.

Combining the solutions from both inequalities, we have -1 ≤ x ≤ 3.

To graph the solution, we plot the interval -1 ≤ x ≤ 3 on the number line. The shaded region on the number line represents the solution set.

To learn more about inequality;

brainly.com/question/28823603

#SPJ1

help me pleaseeeeeeeeeeeeeeee

Answers

I believe it is 160 m/min !

Use the table to add 199+61 vertically. The top row will be for regrouping (numbers that are "carried"). The bottom row will be for your answer. The addends have already been filled in for you.

Answers

The sum of the number 199 + 61 by vertical table will be 260.

Since, Algebra is the study of abstract symbols, while logic is the manipulation of all those ideas.

The acronym PEMDAS stands for Parenthesis, Exponent, Multiplication, Division, Addition, and Subtraction. This approach is used to answer the problem correctly and completely.

Use the table to add 199 + 61 vertically. Then we have

                                Hundreds          Tens          Ones

Regrouping                    1                       1            

First addend                  1                      9               9

Second addend            0                      6               1

Sum                              2                      6               0

Thus, The sum of the number 199 + 61 by the vertical table will be 260.

More about the Algebra link is given below.

brainly.com/question/953809

#SPJ1

The table to add 199+61 vertically is shown below.

To add 199 and 61 vertically, we'll set up the addition table as follows:

 1  9  9

+   6  1

_________

Let's start by adding the digits in the rightmost column, which is the ones place:

9 + 1 = 10

We write down 0 in the bottom row and carry over the 1 to the top row:    1  9  9

+   6  1

_________

   0

Now, let's add the digits in the next column (the tens place), including the carry-over:

1 + 9 + 1 = 11

We write down 1 in the bottom row and carry over the 1 to the top row:

   1  9  9

+   6  1

_________

  1  1

Finally, let's add the digits in the leftmost column (the hundreds place), including the carry-over:

1 + 1 = 2

We write down 2 in the bottom row:

   1  9  9

+   6  1

_________

  2  1

Learn more about Addition here:

https://brainly.com/question/29560851

#SPJ1

determine whether the function represents exponential growth or exponential decay
y=3(1.88)

Answers

Answer:

Exponential Growth

Step-by-step explanation:

The base of 1.88 is greater than 1, so there will be exponential growth

Janet buys 4 bags of potatoes

Answers

what else is there to the question?

People on a specific diet were examined for vitamin B deficiencies. One person is selected at random.


• Let M be the event that the person is deficient in vitamin B,subscript,2,baseline,.


• Let N be the event that the person is deficient in vitamin B,subscript,6,baseline,.

Answers

Option A shows the Venn diagram representing the union of these two events.

What is a Venn Diagram?

A Venn Diagram divides a data-set into separate regions, which are correspond to each activity in the context of the problem.

The intersection region is the region that corresponds to the people sharing both features, with are M and B.

The union region is the region showing at least one of the two features, M and B, hence it is the region in which all the circles are painted.

Hence option a is the correct option in the context of this problem.

More can be learned about Venn Diagrams at brainly.com/question/24713052

#SPJ1

A 12 cm by 12 cm square piece of paper has 5 holes punched out of it. 4 of the holes are circles of radius 3 cm and 1 of the holes is a circle of radius 1 cm. The paper and punched holes can be visually interpreted as below. Determine the area of paper remaining after the holes have been punched out.

Answers

5 holes have been punched into a 12 cm by 12 cm square piece of paper. The area of remaining paper will be 27.714 cm².

Firstly, we will calculate the area of the square of paper in which the holes are punched.

Side of square = 12 cm

Area of square = side²

= (12) ²

= 144 cm²

Now, we will calculate the area of the bigger punch holes

Radius of big punch hole = 3 cm

Area of 1 big punch = π (radius) ²

= 22/7 × (3)²

22 / 7 × 9

= 198 / 7 cm²

Area of 4 punches = 4 × 198/7

= 792/7 cm²

Now, we will calculate the area of smaller punch whose radius is 1cm

Area = 22/7 × 1²

= 22/7 cm²

Now, we will calculate the total area covered by circles

Total area covered by circles = area of small punch + area of 4 big punch

= 22/7 + 792/7

= 814 /7 cm²

Remaining area = area of square - area of circles

= 144 - 814/7

= (1008 - 814) / 7

= 194 / 7

= 27.714 cm²

To know more about area:

https://brainly.com/question/27683633

#SPJ1

A total of $48,000 is invested in two municipal bonds that pay 4.25% and 5.75% simple interest. The investor wants an annual interest income of $2400 from the investments. What amount should be invested in the 4.25% bond

Answers

The amount invested in the 4.25% bond should be $16,000.

please answer i would appreciate it thank you

Answers

a) The triangles are congruent by the AAA criterion.

b) The triangles are congruent by the SAS criterion.

a) In the first triangle the interior angles are 40 degrees and 30 degrees.

The other interior angle would be 180 - (40 + 30) = 110 degrees.

As per the diagram, the triangles have equal proportionate-sized side lengths.

Therefore, the triangles are congruent by the AAA criterion.

b) Both the triangles are right triangles and their adjacent side is 2 and the hypotenuse side is 4 centimeters.

Therefore, the triangles are congruent by the SAS criterion.

To learn more about the congruent;

https://brainly.com/question/12413243

#SPJ1

Tuition for one year at a state university is about $12,500. Greg would like to attend this university and will save money each month for the next 4 years. His parents will give him $4,600 for his first year of tuition. Which plan shows the minimum amount of money Greg must save to have enough money to pay for his first year of tuition?

A. save $658.33 per month for the next 4 years
B. save $383.33 per month for the next 4 year
C. save $260.42 per month for the next 4 years
D. save $164.58 per month for the next 4 years

Answers

Answer: $383.33 per month for the next 4 years

Step-by-step explanation:

The correct answer is save $383.33 per month for the next 4 years. Tuition for one year at a state university is about $12,500, and Greg would like to attend this university. His parents will give him $4,600 for his first year of tuition, so he must save the remaining $7,900. To have enough money to pay for his first year of tuition, Greg must save $383.33 per month for the next 4 years. This is the minimum amount of money he must save, as the other plans require him to save more money than necessary.

Sorry if this is wrong btw fr fr.

Answer:164.58 (D)

Step-by-step explanation:

12,500-4600= 7900

12 months x 4 years makes 48 months, so 7900/48 = 164.58 per month

What is the meaning of "the notational convention"?

Answers

In mathematics, notation is used to simplify and make our understanding of equations and problem statements easier.

Mathematical notation makes use of various symbols in order to represent operations, unspecified numbers, relations, and any other mathematical objects or entities, and proceeds to assemble them into understandable expressions and formulas.

A notational convention is a set of representatives consisting of symbols that are used to express mathematical equations, in this case, the notational convention adopted is that all the free variables of a formula are among u1, u2, u3 . . . un. as shown in the image provided.

To know more about notation, visit :

https://brainly.com/question/30879543

6x + y = 8
y= -6x + 8

Answers

Answer:

y = -6x + 8

Step-by-step explanation:

\begin{bmatrix}6x+y=8\\ y=-6x+8\end{bmatrix}

I don't know why it came out like this

Find the area of the green shaded sector of the circle. Show all of your work. Write your answer in terms of pi.

Answers

The area of the sector with an angle of 285 degrees and a radius of 6 units is 28.5π square units.

To find the area of a sector, we can use the formula:

Area of Sector = (Central Angle / 360 degrees) × π ×  (Radius²)

Given that the central angle is 285 degrees and the radius is 6, we can substitute these values into the formula:

Area of Sector = (285 degrees / 360 degrees) ×  π ×  (6²)

Area of Sector = (19/24) ×  π × 36

Area of Sector = (19/24) ×  36π

Area of Sector = (19 × 36π) / 24

Area of Sector = 684π / 24

Area of Sector = 28.5π

To learn more on Area of Sector click:

https://brainly.com/question/29055300

#SPJ1

B cubed equals 8. Solve for b

Answers

Answer:

b=2

Step-by-step explanation:

You are given:

[tex]b^{3} =8[/tex]

What does [tex]b^3[/tex] mean?

It is simply, b·b·b

Therefore:
b·b·b=8

What number could be multiplied by itself three times and equal 8?

...

2

Yay! That's great!

1.2 Mr John is to deposit R 5,000 in his bank account. Deposits are calculated as follows: Cash deposit : At ATM: R4,80 +1,20% of the above value. At the Branch R8,00 + 1,50% of the above value. John claims that the difference of depositing R5 000 at ATM and at the Branch is R18, 20 Verify the stament (5) [23​

Answers

The calculated difference is -R18.20, which means the deposit at the branch is higher than the deposit at the ATM by R18.20

What is simple interest?

The simple interest formula is J = C ∙ i ∙ t, where J is interest, C is principal, i is interest rate, and t is time. To calculate the simple interest, just substitute the values ​​in the formula and perform the calculation.

Calculate the deposit:

[tex]D=R4.80 + 1.20% of R5,000\\=R4.80 + (1.20/100) * R5,000\\= R4.80 + R60\\= R64.80[/tex]

Now, the deposit amount for the branch:

[tex]DB=R8.00 + 1.50% of R5,000\\= R8.00 + (1.50/100) * R5,000\\= R8.00 + R75\\= R83.00[/tex]

The difference will be:

[tex]Difference = R64.80 - R83.00\\Difference = -R18.20[/tex]

The calculated difference is -R18.20, which means the deposit at the branch is higher than the deposit at the ATM by R18.20. Therefore, John's statement is incorrect.

See more about finances at brainly.com/question/10024737

#SPJ1

please answer i would appreciate it thank you(pls add the reasons)

Answers

The solution is: the value of the angles a, b and c are : a = 60, b = 90, c = 100.

Here,

We have that in line CHF, it has two given and one unknown angle.

The total angle of one straight line is 180.

Given that:

a = 180 - 50 -70 = 60

The angle intersecting the CHFD line can be computed by the the total angle inside the triangle.

Angle on EFH would give 180-40-60 = 80 and on EFD the angle would be 100 from 180-80.

Since angle C is congruent to the angle EFD.

c = 100

Take note that in a 4 sided polygon (GEFH), the total exterior angle is 360 degrees.

The different angle would be

angle GHF = 50 + a = 50+60 = 110

angle EFH = 80

angle FEG = 180- c = 180-100 = 80

ANGLE EGH = 360 - GHF -EFH -FEG = 360 - 110-80-80 = 90

since angle AG is congruent to EGH, we can calculate for the angle b which is congruent to its opposite. This would give us formula of

360 = (90+90) + (b) +(b)

360 = 180 +2b

b = 90

Hence, The solution is: the value of the angles a, b and c are : a = 60, b = 90, c = 100.

To learn more on angle click:

brainly.com/question/28451077

#SPJ1

1. Drag the cards so that each number sentence is true. (You will have one card left over.)
2. Describe your thinking.

Answers

The figures paired are given as follows:

| -3| = 3

|-2| >  1

0 < |-1|

The left over is 2. This is based on the principle of absolute values.

What is absolute values?

The absolute value (or modulus) of a real number x is its non-negative value regardless of its sign. For example, 5 has an absolute value of 5, and 5 has an absolute value of 5.

A number's absolute value can be conceived of as its distance from zero along the real number line.

As a result, the absolute value of -3 is 3, which suggests that |-3| = 3.

-2 in absolute terms equals 2. That is |-2| greater than 1.

the inverse value of -1 equals 1. This equals 0 |-1|.

Learn more about absolute values at:

https://brainly.com/question/31140452

#SPJ1

Tiffany drew the design below that she is going to use on a stained-glass window, above her front door. Identify all the rays in Tiffany’s design

Answers

The rays in Tiffany’s design are EA, FB, GC, GH, ED

Identifying all the rays in Tiffany’s design

From the question, we have the following parameters that can be used in our computation:

Tiffany’s design

By definition, the rays in Tiffany’s design are lines that have one fixed endpoint and the other endpoint extends indefinitely

Using the above as a guide, we have the following:

The rays in Tiffany’s design are EA, FB, GC, GH, ED

Read more about rays at

https://brainly.com/question/13945015

#SPJ1

In circle S, a sector is shaded. ZTSU measures 40°.
C
T
40°
(47)
S
What fraction of the interior of the circle is shaded?
Simplify your answer.
4 cm
Which expression represents the area of the shaded sector in square centimeters
(4x)
(16)
(167)

Answers

Given that in circle S, a sector is shaded and ZTSU measures 40°. We are to find out what fraction of the interior of the circle is shaded and simplify the answer.The correct answer is 16π/9 cm².

To find out the fraction of the interior of the circle that is shaded, we need to determine the measure of the central angle that intercepts the entire circle.The total measure of a circle is 360 degrees.

To find the fraction of a circle shaded, divide the number of degrees in the sector by

360.4x = (40/360) × πr² = (1/9) × πr².

We simplify the answer: 4x = πr²/9 Fraction of the interior of the circle that is shaded = 40/360 = 1/9.Expression for the area of the shaded sector in square centimeters is:

A = (40/360) × πr² = πr²/9 =  π(4²)/9 = 16π/9 cm²

Therefore, the correct answer is 16π/9 cm².

For more such questions on circle

https://brainly.com/question/28162977

#SPJ8

A fish tank holds 75 gallons of water and has a density of 0.12 fish per gallon. How many. How many fish are in the fish tank

Answers

Answer: 9 fish

Step-by-step explanation:

To find the answer all you have to do is multiply the amount of fish per gallon (0.12) to the total amount to gallons (75).

75 x 0.12 = 9

A 75-gallon aquarium will hold about 8 fish.

Stocking guidelines are 1 inch of adult fish per gallon of tank water. When applying this rule of thumb, we have to keep in mind that a 75 gallons water tank only holds about 70 gallons of water.

Then the number of fish in the tank would be:

Number of fish = density of fish * volume of water in the tank

Number of fish = 0.12 * 70

Number of fish = 8.4

So there are about 8 fish in the tank.

To learn more about Density and Volume,

https://brainly.com/question/1568313

https://brainly.com/question/95059

Other Questions
A sample of flammable liquid is placed into an enclosed cylinder which is then fitted with a movable piston. Initially the cylinder maintains a volume of 1.7 L. The sample is ignited producing gas and releasing 439.6 J of energy. To what volume will the cylinder expand to if it must expand against a pressure of 738.52 mmHg. Assume all the energy released is converted to work used to push the piston? Answer to 1 decimal space. Instructions: Find the measure of the indicated angle to the nearest degree. A nurse is caring for a client following an abdominal surgery. The client has a Penrose drain in place under the surgical dressing. The dressing is to be changed every 4 hr and as needed. Which of the following objects should the nurse use to reduce skin irritation around the incision area The increased importance that politicans have placed on public opinion polls over the last 50 years is:______ A car manufacturer is reducing the number of incidents with the transmission by issuing a voluntary recall. during week 10 of the recall, the manufacturer fixed 200 cars. in week 15, the manufacturer fixed 175 cars. assume that the reduction in the number of cars each week is linear. write an equation in function form to show the number of cars seen each week by the mechanic. f(x) = 5x 250 f(x) = 5x 250 f(x) = 10x 200 f(x) = 10x 200 A store is having a sale on chocolate chips and walnuts. For 5 pounds of chocolate chips and 3 pounds of walnuts, the total cost is $25 . For 7 pounds of chocolate chips and 9 pounds of walnuts, the total cost is $53 . Find the cost for each pound of chocolate chips and each pound of walnuts. For the government to restrict speech, the imminent lawless action test requires ______. Select the correct answer. Which organization is an interest group Identify the major species, other than water and potassium ions, at these points Describe a situation where you provided or observed high-quality customer service. in shipt (x-1)^2 + (y-2)^2 = 6.25 Consider the dot plot below. Of the following statements, which two characteristics of this dot plot make the median a better choice than the mean to summarize the center of the distribution?A dot plot with an axis marked from 0 to 10 at increments of 1 is shown. Plot shows 8 dots at 10, 7 dots at 9, 6 dots at 8, 3 dots at 7, 2 dots at 0 and 6, and 1 dot at 5. The mean is equal to the median and the data are symmetric. The peak is equal to the median and the data are skewed. The data are symmetric and there are outliers. The data are skewed and there are outliers. g Why is a vegetarian diet ecologically and energetically less expensive than a diet that includes meat what do multinational corporations needs to balance when making decisions about social responsibilities? John walks 3\, \text m3m3, start text, m, end text east and 4\, \text m4m4, start text, m, end text north. What is another name for XW?Name a ray with an endpoint X.Give another name for WX.Whats another name for AXW?Double points Six men, A, B, C, D, E, and F of negligible honesty, met on a perfectly rough day, each carrying a light inextensible umbrella. Each man brought his own umbrella and took away let us say 'borrowed' _ - another's. The umbrella borrowed by A belonged to borrower of B's umbrella. The owner of the umbrella borrowed by C borrowed the umbrella belonging to the borrower of D's umbrella. If the borrower of E's umbrella was not the owner of that borrowed by F, who borrowed A's umbrella? Even when you have the right-of-way, you're responsible forbefore youdrive forward. Some bacteria double in every hour. if there were 2 bacteria in the beginning, how many bacteria will be there after 6 hours? how many bacteria will be there after 24 hours? Solve the following 8903=e